Đến nội dung

Math04 nội dung

Có 119 mục bởi Math04 (Tìm giới hạn từ 29-04-2020)



Sắp theo                Sắp xếp  

#742253 Chứng minh $SN \perp BC$

Đã gửi bởi Math04 on 26-11-2023 - 22:40 trong Tài liệu, chuyên đề, phương pháp về Hình học

 

attachicon.gif screenshot_1700968049.png

Ta có: $\angle SNB=\angle SAX-\angle XAB=\frac{1}{2}\angle XAY-\angle NAB=\angle BAC-\angle NAB=\angle NAC$

Suy ra $AN$, $AS$ đẳng giác trong góc $A$ nên $AS$ đi qua tâm $Z$ của $(BOC)$ với $O$ là tâm $(ABC)$

Gọi $H$ là trực tâm tam giác $ABC$

Ta có:

$BS^{2}=\frac{BA^{2}}{2}+\frac{BZ^2}{2}-\frac{AZ^2}{4}$

$CS^{2}=\frac{CA^{2}}{2}+\frac{CZ^2}{2}-\frac{AZ^2}{4}$

$BN^{2}=\frac{BH^2}{2}+\frac{BO^2}{2}-\frac{OH^2}{4}$

$CN^{2}=\frac{CH^2}{2}+\frac{CO^2}{2}-\frac{OH^2}{4}$

Suy ra:

$(BS^2-BN^2)-(CS^2-CN^2)=(BA^2-BH^2)-(CA^2-CH^2)=0$

Áp dụng định lí 4 điểm ta có luôn $SN$ vuông góc $BC$ (dpcm)

 

 Bạn ơi cho mình hỏi làm sao chứng minh $Z$ thuộc $(AXY)$ nhỉ




#742239 $d(X,AB)+d(X,AC)=d(X,BC)$

Đã gửi bởi Math04 on 26-11-2023 - 17:14 trong Hình học

Cho tam giác $ABC$ với $AB<AC$. Phân giác trong $BE, CF$, $E,F$ thuộc $AC, AB$. Với $X$ bất kỳ trong tam giác, chứng minh $X$ thuộc đoạn $EF$ khi và chỉ khi $d(X,AB)+d(X,AC)=d(X,BC)$, với $d(X,AB)$ là khoảng cách từ $X$ đến $AB$. Tương tự cho $BC, AC$.




#742219 Chứng minh $SN \perp BC$

Đã gửi bởi Math04 on 25-11-2023 - 16:31 trong Tài liệu, chuyên đề, phương pháp về Hình học

Cho tam giác $ABC$ nội tiếp $(O)$, gọi $N$ là tâm đường tròn Euler. Gọi $X,Y$ lần lượt đối xứng với $N$ qua $AB, AC$. $S$ là tâm $(AXY)$. Chứng minh $SN \perp BC$.




#738047 $\frac{a^2+b^2+c^2}{ab+bc+ca}+\frac{8...

Đã gửi bởi Math04 on 25-03-2023 - 17:43 trong Bất đẳng thức - Cực trị

Việc hỏi bài trên diễn đàn không có gì sai nhé các bạn, cứ mạnh dạn đăng bài nếu cần sự giúp đỡ. Nhưng cũng có thể @chuyenndu không phải chỉ trích gì mà chỉ có ý khuyến khích @Math04 tham gia thảo luận hơn mà thôi, em nên xem đó như là một lời góp ý.

 

Việc học trên diễn đàn nếu chỉ hỏi bài thôi thì hơi uổng. @Math04 thử tìm những bài chưa giải (có rất nhiều) và thử giải xem sao, anh nghĩ là có nhiều bài vừa sức với em đấy, như thế sẽ học được nhiều hơn. Ngoài ra lúc em hỏi bài, thì cũng có thể trình bày xem là mình đã cố gắng làm như thế nào, bí chỗ nào, như thế thì mọi người có thể gợi ý cho em tự làm tiếp, sẽ hiệu quả hơn cho em đấy. Có thể thử bắt đầu với bài ở trên chẳng hạn  :like

Em cám ơn anh ạ




#738044 $\frac{a^2+b^2+c^2}{ab+bc+ca}+\frac{8...

Đã gửi bởi Math04 on 25-03-2023 - 17:07 trong Bất đẳng thức - Cực trị

mình thấy bạn này chỉ toàn hỏi bài nhỉ?

Mình thắc mắc nên mới hỏi thôi bạn, với mình cũng không chuyên lắm nên mình cũng đâu đi giải bài khác được bạn. Mình nghĩ diễn đàn lập ra để trao đổi, những người chưa biết thì phải hỏi thôi bạn?




#738040 $\frac{a^2+b^2+c^2}{ab+bc+ca}+\frac{8...

Đã gửi bởi Math04 on 25-03-2023 - 15:51 trong Bất đẳng thức - Cực trị

Cho $a,b,c>0$. Dùng phương pháp $p,q,r$ hãy chứng minh: $\frac{a^2+b^2+c^2}{ab+bc+ca}+\frac{8abc}{(a+b)(b+c)(c+a)}\geq2$




#738039 Tính $a_n$ là số cách điền thỏa ba ô liên tiếp nhau bất kỳ đều khôn...

Đã gửi bởi Math04 on 25-03-2023 - 15:19 trong Tổ hợp và rời rạc

Ta gọi một cách điền "rất đẹp" nếu như đó là cách điền đẹp và có hai ô cuối cùng có số giống nhau.

Thế thì gọi $b_n$ là số cách điền "rất đẹp".

+) Tính $a_{n+1}$: Với mỗi cách điền $n$ ô trước là "đẹp" nhưng không phải "rất đẹp" ta có $2$ cách điền ô thứ $n+1$. Đồng thời với mỗi cách điền $n$ ô trước "không đẹp" ta có duy nhát $1$ cách điền ô thứ $n=1$. Do đó $a_{n+1} = 2a_n - b_n$.

+) Tính $b_{n+1}$: Giả sử ô thứ $n$ và $n+1$ đều là $0$. Thế thì ô thứ $n-1$ là $1$, và số cách điền lúc này chính là số cách điền $n-2$ ô đầu tiên có hai vị trí cuối không cùng bằng $1$, và bằng $a_{n-2} - \frac{b_{n-2}}{2}$.

Suy ra $b_{n+1} = 2a_{n-2} - b_{n-2}$.

Kết hợp lại ta có công thức truy hồi: $\begin{cases} a_{n+1} = 2a_n - b_n \\ b_{n+1} = 2a_{n-2} - b_{n-2}\end{cases}$

$\Rightarrow b_n = 2a_n - a_{n+1},\forall n\geq 1\Rightarrow 2a_{n+1} - a_{n+2} = 2a_{n-2} - 2a_{n-2} + a_{n-1},\forall n\geq 3$

$\Rightarrow a_{n+2} = 2a_{n+1} - a_{n-1},\forall n\geq 3$.

Tính được: $a_1 = 2, a_2 = 4, a_3 = 6, a_4 = 10$. Do đó ta có $a_{n+3} = 2a_{n+2} - a_n,\forall n\in\mathbb N^*$.

Từ đây dễ dàng tìm được CTTQ của $(a_n)$.

Hoàng có thể chia sẻ cách em học các phân môn không nhỉ cũng như từng tài liệu/ nguồn bài mà em tham khảo




#737953 Tính $a_n$ là số cách điền thỏa ba ô liên tiếp nhau bất kỳ đều khôn...

Đã gửi bởi Math04 on 23-03-2023 - 10:43 trong Tổ hợp và rời rạc

Xét hình chữ nhật $1 \times n$ gồm $n$ ô vuông $1 \times 1$. Mỗi ô điền số $0$ hoặc $1$. Một cách điền "đẹp" nếu như ba ô liên tiếp nhau bất kỳ đều không chứa ba số giống nhau. Với mỗi $n \geq 3$, gọi $a_n$ là số cách điền "đẹp". Tính $a_n$.




#737951 Chứng minh dù ban đầu các đống sỏi sắp xếp như thế nào, sau hữu hạn bước luôn...

Đã gửi bởi Math04 on 23-03-2023 - 10:34 trong Tổ hợp và rời rạc

Có $20$ viên sỏi xếp thành ba đống. Mỗi lần cho phép lấy một nửa số sỏi từ đống có chẵn viên sỏi và chuyển sang đống khác. Chứng minh dù ban đầu các đống sỏi sắp xếp như thế nào, sau hữu hạn bước luôn có thể tạo ra một đống sỏi chứa đúng $10$ viên sỏi.




#737935 Đặt $b_n=\sqrt{n}(a_n-L)$ với $L=lima_n$....

Đã gửi bởi Math04 on 23-03-2023 - 00:00 trong Dãy số - Giới hạn

Với số thực $x$ có phần lẻ khác $\frac{1}{2}$, ta kí hiệu $<x>$ là số nguyên gần nhất với $x$. Xét dãy: $a_n=\sum_{k=1}^{n}\frac{1}{<\sqrt{k}>}-2\sqrt{n}, n=1,2...$.

a) Chứng minh dãy trên hội tụ và tìm giới hạn đó.

b) Đặt $b_n=\sqrt{n}(a_n-L)$ với $L=lima_n$. Chứng minh với mọi số thực $\alpha \in [0;\frac{1}{4}]$, luôn tồn tại một dãy con của $(b_n)$ có giới hạn bằng $\alpha$




#737934 Cho $n \in \mathbb{N}$ hãy chứng minh rằng tồn...

Đã gửi bởi Math04 on 22-03-2023 - 23:52 trong Số học

Cho $n \in \mathbb{N}$ hãy chứng minh rằng tồn tại số tự nhiên $k$ sao cho với mọi $m\ge k$ , tồn tại một dãy $m$ số tự nhiên liên tiếp mà chứa đúng $n$ số nguyên tố.




#737933 Chứng minh rằng với $n \in \mathbb{N}$ mọi số nguyên dươn...

Đã gửi bởi Math04 on 22-03-2023 - 23:47 trong Số học

Gọi $p_k$ là số nguyên tố thứ $k$ và gọi $a_n=\prod_{k=1}^{n}p_k$. Chứng minh rằng với $n \in \mathbb{N}$ mọi số nguyên dương nhỏ hơn $a_n$ đều có thể biểu diễn dưới dạng tổng của tối đa $2n$ ước phân biệt của $a_n$.




#737469 Tìm giá trị lớn nhất của $a$ để $x_{2023}=0$

Đã gửi bởi Math04 on 27-02-2023 - 21:07 trong Dãy số - Giới hạn

Cho số thực $a>0$ , xét dãy:
$$\left\{\begin{matrix} \left(k+1\right)x_{k+2}=ax_{k+1}+\left(k-2022\right)x_k & & \\ x_0=0, x_1=1 & & \end{matrix}\right.$$

Tìm giá trị lớn nhất của $a$ để $x_{2023}=0$




#737468 $lim\frac{x_1+...+x_n}{n\sqrt{x_n}...

Đã gửi bởi Math04 on 27-02-2023 - 20:54 trong Dãy số - Giới hạn

Cho dãy dương $(x_n)$, tăng ngặt, $limx_n=+\infty$. Đặt $v_n=x_{n+1}-x_{n}$. Biết $(v_n)$ đơn điệu. Tính $lim\frac{x_1+...+x_n}{n\sqrt{x_n}}$




#736843 Chứng minh dãy $(x_n)$ hội tụ với $x_{n+2}+x_{n...

Đã gửi bởi Math04 on 18-01-2023 - 22:39 trong Dãy số - Giới hạn

Cho dãy $(x_n)$ bị chặn trên và dưới đồng thời thỏa: $\left\{\begin{matrix}x_1=3,x_2=1 &  & \\ x_{n+2}+x_{n}<2x_{n+1}+\frac{1}{n^2}, \forall n=1,2,3,... &  & \\ \end{matrix}\right.$. Chứng minh dãy $(x_n)$ hội tụ.




#736842 Chứng minh $lima_n=limb_n=limc_n=+\infty$ biết $a_{n...

Đã gửi bởi Math04 on 18-01-2023 - 22:35 trong Dãy số - Giới hạn

Cho ba dãy dương $(a_n),(b_n),(c_n)$ thỏa $\forall n \geq 0$: $\left\{\begin{matrix}a_{n+1}=a_n+\frac{1}{\sqrt{b_nc_n}} &  & \\b_{n+1}=b_n+\frac{1}{\sqrt{c_na_n}} &  & \\ c_{n+1}=c_n+\frac{1}{\sqrt{a_nb_n}} &  & \end{matrix}\right.$ Chứng minh $lima_n=limb_n=limc_n=+\infty$




#736841 Chứng minh dãy $(s_n)$ với $s_n=x_1+x_2+...+x_n$ hội tụ

Đã gửi bởi Math04 on 18-01-2023 - 22:31 trong Dãy số - Giới hạn

Cho dãy $(x_n)$ là dãy dương thỏa: $lim\frac{x_{n+1}}{x_n}=\frac{1}{2}$. Chứng minh dãy $(s_n)$ với $s_n=x_1+x_2+...+x_n$ hội tụ.




#736738 Xác định số nghiệm thực của hệ

Đã gửi bởi Math04 on 12-01-2023 - 21:48 trong Phương trình - Hệ phương trình - Bất phương trình

Cho trước số thực $k>9$. Xét hệ sau với $a\leq b\leq c\leq d$:

$\left\{\begin{matrix} a+b+c+d=k+3\\ a^2+b^2+c^2+d^2=k^2+3\\ abcd=k \end{matrix}\right.$

a) Tìm tất cả các nghiệm trong tập số dương.
b) Xác định số nghiệm thực của hệ.



#736640 Chứng minh tồn tại số thực $c > 0$ sao cho $P(x)$ đơn...

Đã gửi bởi Math04 on 05-01-2023 - 22:58 trong Đa thức

Cho đa thức $P(x)$ hệ số thực, có bậc không nhỏ hơn $1$. 

a) Khi đó $\forall \varepsilon >0$, chứng minh tồn tại $c$ sao cho $|P(x)| > c, \forall |x| > c$. 

b) Chứng minh tồn tại số thực $c > 0$ sao cho $P(x)$ đơn điệu trên $(−∞, −c)$ và $(c, +∞)$




#736603 $f^2(x)+f^2(y) \leq 2f(xy), \forall x,y \in \mathbb...

Đã gửi bởi Math04 on 01-01-2023 - 22:29 trong Phương trình hàm

Tìm $f:\mathbb{R}\rightarrow \mathbb{Z}$ thỏa:

$f^2(x)+f^2(y) \leq 2f(xy), \forall x,y \in \mathbb{R}$.




#736601 $xf\left(x+y\right)\ge\left(y+1\right)f\le...

Đã gửi bởi Math04 on 01-01-2023 - 22:21 trong Phương trình hàm

Tìm $f:\mathbb{R}\rightarrow \mathbb{R}$ thỏa:

$xf\left(x+y\right)\ge\left(y+1\right)f\left(x\right)+x-1, \forall x,y \in \mathbb{R} $




#736596 Chứng minh tồn tại số nguyên dương $M$ sao cho: $a_{m...

Đã gửi bởi Math04 on 01-01-2023 - 17:25 trong Số học

Bạn có kinh nghiệm gì khi mà tìm bài trên AOPS không nhỉ kiểu gõ công thức hay là gõ đề vào tại nhiều lúc mình tìm mà không ra. Với tìm ngay chỗ Search Community hay là Advanced Community Search nhỉ




#736579 Chứng minh tồn tại số nguyên dương $M$ sao cho: $a_{m...

Đã gửi bởi Math04 on 31-12-2022 - 21:19 trong Số học

Đặt $s_n = \frac{a_1}{a_2} + \frac{a_2}{a_3}+...+\frac{a_{n-1}}{a_n}+\frac{a_n}{a_1}$.

Ta có $s_n\in\mathbb Z,\forall n\geq N$ nên $s_{n+1} - s_n\in\mathbb Z,\forall n\geq N$

$\Rightarrow \frac{a_n}{a_{n+1}} + \frac{a_{n+1}}{a_1} - \frac{a_n}{a_1}\in\mathbb Z,\forall n\geq N$.

Lấy $p$ là số nguyên tố bất kì. 

Nhận xét: Với mọi $n\geq N$: Nếu $v_p(a_n)> v_p(a_1)$ thì $v_p(a_{n+1})> v_p(a_1)$. Ngược lại, nếu $v_p(a_n)\leq v_p(a_1)$ thì $v_p(a_{n+1})\leq v_p(a_1)$

Chứng minh: Giả sử phản chứng.

TH1: $v_p(a_n)> v_p(a_1)$: Khi đó theo giả sử thì $v_p(a_{n+1}) \leq v_p(a_1) < v_p(a_n)$

$\Rightarrow v_p\left(\frac{a_{n+1}}{a_1}\right) < v_p\left(\frac{a_n}{a_1}\right) \leq v_p\left(\frac{a_n}{a_{n+1}}\right)$

$\Rightarrow v_p\left(\frac{a_n}{a_{n+1}} + \frac{a_{n+1}}{a_1} - \frac{a_n}{a_1}\right) = v_p\left(\frac{a_{n+1}}{a_1}\right) < 0$, vô lí vì $\frac{a_n}{a_{n+1}} + \frac{a_{n+1}}{a_1} - \frac{a_n}{a_1}\in\mathbb Z$.

TH2: $v_p(a_n) \leq v_p(a_1)$: Khi đó theo giả sử thì $v_p(a_n) \leq v_p(a_1) < v_p(a_{n+1})$

$\Rightarrow v_p\left(\frac{a_n}{a_{n+1}}\right) < v_p\left(\frac{a_n}{a_1}\right) <v_p\left(\frac{a_{n+1}}{a_1}\right)$

$\Rightarrow v_p\left(\frac{a_n}{a_{n+1}} + \frac{a_{n+1}}{a_1} - \frac{a_n}{a_1}\right) = v_p\left(\frac{a_n}{a_{n+1}}\right) < 0$, vô lí vì $\frac{a_n}{a_{n+1}} + \frac{a_{n+1}}{a_1} - \frac{a_n}{a_1}\in\mathbb Z$.

Từ nhận xét trên, ta thấy $v_p(a_n) > v_p(a_1),\forall n\geq N$ hoặc $v_p(a_n) \leq v_p(a_1),\forall n \geq N$.

$\bullet$ Nếu $v_p(a_n) > v_p(a_1),\forall n\geq N$ thì $v_p(a_n)\geq v_p(a_{n+1}),\forall n\geq N$.

Thật vậy, do $v_p\left(\frac{a_{n+1}}{a_1} - \frac{a_n}{a_1}\right) >0,\forall n\geq N$ và $v_p\left(\frac{a_n}{a_{n+1}} + \frac{a_{n+1}}{a_1} - \frac{a_n}{a_1}\right),\forall n\geq N$ nên $v_p\left(\frac{a_n}{a_{n+1}}\right)\geq 0,\forall n\geq N$

$\Rightarrow v_p(a_n)\geq v_p(a_{n+1}),\forall n\geq N$.

$\bullet$ Nếu $v_p(a_n)\leq v_p(a_1),\forall n\geq N$ thì ta vẫn có $v_p(a_n)\geq v_p(a_{n+1}),\forall n\geq N$.

Thật vậy, giả sử tồn tại $n\geq N$ sao cho $v_p\left(a_{n+1}\right) > v_p\left(a_{n}\right)$.

Thế thì $v_p\left(\frac{a_{n+1}}{a_1}\right) < v_p\left(\frac{a_n}{a_1}\right) \leq v_p\left(\frac{a_n}{a_{n+1}}\right)\Rightarrow v_p\left(\frac{a_n}{a_{n+1}} + \frac{a_{n+1}}{a_1} - \frac{a_n}{a_1}\right) = v_p\left(\frac{a_{n+1}}{a_1}\right) < 0$, vô lí.

Do đó dãy $u_n = v_p(a_n)$ là dãy không tăng kể từ $N$. Mặt khác, dãy này là dãy số nguyên không âm nên đến một lúc nào đó, dãy là hằng.

Điều này đúng với mọi số nguyên tố $p$ nên suy ra $(a_n)$ là hằng số kể từ chỉ số nào đó. (đpcm)

Cám ơn bạn nhé làm sao để nghĩ ra lời giải như vầy nhỉ




#736575 Chứng minh rằng tồn tại số nguyên $a$, $1<a<\frac...

Đã gửi bởi Math04 on 31-12-2022 - 14:02 trong Số học

Bài này nhìn tưởng đơn giản nhưng để thỏa mãn điều kiện bất đẳng thức thì rắc rối ghê  :wacko:

 

Đặt phân tích ra thừa số nguyên tố của $n$ là $p_1^{\alpha_1}p_2^{\alpha_2}\dots p_k^{\alpha_k}$. Theo định lí thặng dư Trung Hoa, với mỗi $i\in \{1,2,\dots,k\}$ thì tồn tại số nguyên $x_i$ thỏa mãn

\[\left\{\begin{array}{l}x_i\equiv 1\pmod{p_i^{\alpha_i}}\\ x_i\equiv 0\pmod{\frac{n}{p_i^{\alpha_i}}}\end{array}\right..\]

Nhận xét (NX). Cho tổng $X=\sum_{i=1}^k\epsilon_ix_i$, trong đó $\epsilon_i\in \{0,1\}$ với mọi $i$.

  1. $n\mid X^2-X$.
  2. Nếu $X$ chia $n$ dư $0$ hoặc $1$ thì $\epsilon_1=\epsilon_2=\dots=\epsilon_k$.

 

Với mỗi $i\in \{1,2,\dots,k\}$, gọi $X_i$ là số tự nhiên nhỏ nhất thỏa mãn

\[X_i\equiv x_1+x_2+\dots+x_i\pmod{n}.\]

Theo NX2 thì $X_i\neq 0$ với mọi $i=\overline{1,k}$ và ngoài ra $X_k=1$. Bổ sung thêm $X_0=0$. Xét $k+1$ số $X_0,\ X_1,\ \dots,\ X_k$, theo định lí Dirichlet sẽ tồn tại $0\le c<d\le k$ sao cho

\[X_c,\ X_d\in\left [ \frac{jn}{k},\frac{(j+1)n}{k} \right ).\]

Nghĩa là $|X_d-X_c|<\frac{n}{k}$, theo NX2 ta có $X_d-X_c\notin \{0,1\}$. Cuối cùng ta chia ra hai trường hợp sau

$\bullet$ TH1: $X_d-X_c>1$ thì đặt

$$a_1=x_{c+1}+x_{c+2}+\dots+x_d=X_d-X_c\in\left(1,\frac{n}{k}\right).$$

Ngoài ra theo NX1 thì $n\mid a_1^2-a_1$ nên $a_1$ thỏa đề.

$\bullet$ TH2: $X_d-X_c<0$ thì đặt

$$a_2=(x_1+x_2+\dots+x_c)+(x_{d+1}+x_{d+2}+\dots+x_k)=X_k-(X_d-X_c)\in\left(1,1+\frac{n}{k}\right).$$

Ngoài ra theo NX1 thì $n\mid a_2^2-a_2$ nên $a_2$ thỏa đề.

À bạn giải thích rõ chỗ "theo định lí Dirichlet sẽ tồn tại $0\le c<d\le k$ sao cho

\[X_c,\ X_d\in\left [ \frac{jn}{k},\frac{(j+1)n}{k} \right ).\]" giúp mình với. Bạn áp dụng nguyên lí định lí Dirichlet như thế nào nhỉ




#736508 Chứng minh có thể chọn ra một vài số trong $n$ số này mà có tổng bằ...

Đã gửi bởi Math04 on 30-12-2022 - 10:57 trong Tổ hợp và rời rạc

Cho $S$ là một bội nguyên dương của tất cả các số từ $2$ đến $2019$ và $n$ số nguyên dương $a_{1},...,a_{n}$ thuộc $M=\left \{ 1,2,...,2019 \right \}$ có tổng bằng $2S$. Chứng minh có thể chọn ra một vài số trong $n$ số này mà có tổng bằng $S$.